Quora's productivity

This topic has expert replies
User avatar
Senior | Next Rank: 100 Posts
Posts: 76
Joined: Mon Jan 20, 2014 3:24 am
Thanked: 3 times

Quora's productivity

by Amadalia » Mon Apr 28, 2014 3:50 am
Because of the recent transformation of the market, Quore Inc. must increase
productivity 10 percent over the course of the next two years or it will certainly go
bankrupt. In fact, however, Quore's production structure is such that if a 10 percent
productivity increase is possible, then a 20 percent increase is attainable.
If the statements above are true, which one of the following must on the basis of
them also be true?
(A) It is only Quore's production structure that makes it possible fro Quore to
survive the transformation of the market.
(B) Quore will not go bankrupt if it achieves a productivity increase of 20 percent
over the next two years.
(C) If the market had not been transformed, Quore would have required no
productivity increase in order to avoid bankruptcy.
(D) Because of the transformation of the market Quore will achieve a productivity
increase of 10 percent over the next two years.
(E) If a 20 percent productivity increase is unattainable for Quore, then it must go


My question is the following :
According to the stimilus : Recent transformation ------->Quora must increase the productivity

C says: if recent transformation doesnt happen ------->no increase in Qura's production would be necessary
x---->y
not x --->not y
what is wrong with C?
Many thanks in advance
bankrupt.

GMAT/MBA Expert

User avatar
Master | Next Rank: 500 Posts
Posts: 166
Joined: Thu Dec 12, 2013 3:17 pm
Location: Berkeley, Ca
Thanked: 30 times
Followed by:21 members

by KevinRocci » Mon Apr 28, 2014 9:34 am
Happy to help! Let's start by breaking down the argument.

Premise: market change
Premise: Quore increase productivity 10%
Premise: increase over 2 years
Premise: no increase = bankruptcy
Premise: If 10% possible, then 20% possible

Alright, that is the basic structure of our argument. Now we can look at your question.

NOw, I can see why you were tempted by (C). You reasoning being:

x -> y
no x -> no y

But the important point to realize is that there might be other reasons that Quore would go bankrupt. They might have a disasters marketing campaign. There might be new competitors in the market. The company might have invested in a risky technology. There might be an economic recession.

I know that all these points are speculation, but it is important to realize that this company might still have a threat of bankruptcy. There is nothing in the argument that would allow us to say that Quore would not have to be more productivity to avoid bankruptcy. We are only talking about one possible case for increasing productivity. There might be others that we don't know about.

We need to choose an answer that makes no assumptions and fits neatly into the premises that we already have in the argument.

I hope that this helps! :)

User avatar
Senior | Next Rank: 100 Posts
Posts: 76
Joined: Mon Jan 20, 2014 3:24 am
Thanked: 3 times

by Amadalia » Mon Apr 28, 2014 9:40 am
Oh yes It perfectly does
X---->y
not X----->Y may happen or may not happen

Many thanks Kevin!!!!

User avatar
Newbie | Next Rank: 10 Posts
Posts: 6
Joined: Wed Oct 09, 2013 3:54 am
Thanked: 1 times

by nandini.gaur » Wed Apr 30, 2014 2:13 am
HI

I think the answer is B.. am i correct?

User avatar
Senior | Next Rank: 100 Posts
Posts: 76
Joined: Mon Jan 20, 2014 3:24 am
Thanked: 3 times

by Amadalia » Wed Apr 30, 2014 2:26 am
at all,
B is too extreme, it says that the company's will not go bunkrupt if it realise 20% increase in productivity
First it's not what the stimilus said and second even is the company can reach that level it may be unwise to do it imagine that the maket can not absorb such quantities?

hope this help

GMAT/MBA Expert

User avatar
GMAT Instructor
Posts: 645
Joined: Mon Jul 26, 2010 12:42 pm
Location: US
Thanked: 527 times
Followed by:227 members

by e-GMAT » Thu May 08, 2014 11:04 pm
nandini.gaur wrote:HI

I think the answer is B.. am i correct?
Hi Nandini,

You are not correct. I think where you are going wrong is a confusion between necessary and sufficient condition.

The passage says:

Quore Inc. must increase productivity 10 percent over the course of the next two years or it will certainly go bankrupt.

So, it lays down a necessary condition to be fulfilled to avoid bankruptcy. Basically, it says:

You must do X in order to avoid Y.

Now, option B says:

Quore will not go bankrupt if it achieves a productivity increase of 20 percent over the next two years.

Y will be avoided if X is done. (I am ignoring the difference "10% vs 20%" in passage and option here)

Now, option B states that X is sufficient to ensure that Y is avoided. So, it says X is a sufficient condition for avoidance of Y.

While per the passage, even if X was done, there was a possibility that Y could not have been avoided. Suppose some other Z was also required to avoid Y and Z was not done. In such a case, Y would not have happened. So, from the passage, there existed a possibility that even when X was done, Y may not happen.

This possibility does not exist in option B. In this case, Y will necessarily be avoided if X is done.

In essence, a common confusion of necessary and sufficient condition.

I hope it helps!

Thanks,
Chiranjeev

GMAT/MBA Expert

User avatar
Elite Legendary Member
Posts: 10392
Joined: Sun Jun 23, 2013 6:38 pm
Location: Palo Alto, CA
Thanked: 2867 times
Followed by:511 members
GMAT Score:800

by [email protected] » Thu May 08, 2014 11:59 pm
Hi Amadalia,

What is the source of this question? While the logic behind this question is something that you might see on the GMAT, the wording/design of both the prompt and the answer choices is not.

GMAT assassins aren't born, they're made,
Rich
Contact Rich at [email protected]
Image

Junior | Next Rank: 30 Posts
Posts: 15
Joined: Wed Jul 03, 2013 5:28 am
Thanked: 3 times

by moumi2013 » Thu May 15, 2014 5:49 am
Someone please provide the OA.
my answer was B but from the given explanation it looks like B is not the correct answer.

Moreover is this the comeplete statement for E :
(E) If a 20 percent productivity increase is unattainable for Quore, then it must go

Newbie | Next Rank: 10 Posts
Posts: 9
Joined: Mon Jun 02, 2014 8:19 am

by appy_fizz » Sat Jun 07, 2014 7:28 am
Can anybody help with the OA. In my view the OA should be D. Please help.